Difference between revisions of "2025 AMC 8 Problems"

m (Disclaimer)
m (Several warnings)
Line 6: Line 6:
  
 
==Problem 2==
 
==Problem 2==
 +
 +
The 2025 AMC 8 is not held yet. '''Please do not post false problems.'''
 +
 +
==Problem 3==
 +
 +
The 2025 AMC 8 is not held yet. '''Please do not post false problems.'''
 +
 +
==Problem 4==
 +
 +
The 2025 AMC 8 is not held yet. '''Please do not post false problems.'''
 +
 +
==Problem 5==
  
 
The 2025 AMC 8 is not held yet. '''Please do not post false problems.'''
 
The 2025 AMC 8 is not held yet. '''Please do not post false problems.'''

Revision as of 08:53, 18 February 2024

Problem 1

Let $m$ and $n$ be $2$ integers such that $m>n$. Suppose $m+n=20$, $m^2+n^2=328$, find $m^2-n^2$.

$\textbf{(A)}\ 280 \qquad \textbf{(B)}\ 292 \qquad \textbf{(C)}\ 300 \qquad \textbf{(D)}\ 320 \qquad \textbf{(E)}\ 340$

Problem 2

The 2025 AMC 8 is not held yet. Please do not post false problems.

Problem 3

The 2025 AMC 8 is not held yet. Please do not post false problems.

Problem 4

The 2025 AMC 8 is not held yet. Please do not post false problems.

Problem 5

The 2025 AMC 8 is not held yet. Please do not post false problems.